Difference between revisions of "2021 Fall AMC 10A Problems/Problem 1"

m
(redirect to 12A)
(Tag: New redirect)
Line 1: Line 1:
{{duplicate|[[2021 Fall AMC 10A Problems#Problem 1|2021 AMC 10B #1]] and [[2021 Fall AMC 10A Problems#Problem 1|2021 AMC 12B #1]]}}
+
#REDIRECT [[2021 AMC 12A Problems/Problem 1]]
 
 
== Problem ==
 
 
 
What is the value of <math>\frac{(2112-2021)^2}{169}</math>?
 
 
 
<math>\textbf{(A) } 7 \qquad\textbf{(B) } 21 \qquad\textbf{(C) } 49 \qquad\textbf{(D) } 64 \qquad\textbf{(E) } 91</math>
 
 
 
== Solution ==
 
We have <cmath>\frac{(2112-2021)^2}{169}=\frac{91^2}{169}=\frac{91^2}{13^2}=\left(\frac{91}{13}\right)^2=7^2=\boxed{\textbf{(C) } 49}.</cmath>
 
~MRENTHUSIASM
 
 
 
==See Also==
 
{{AMC10 box|year=2021 Fall|ab=A|before=First Problem|num-a=2}}
 
{{MAA Notice}}
 

Revision as of 18:22, 23 November 2021